MBE Contracts Mistake Books

Réussis tes devoirs et examens dès maintenant avec Quizwiz!

A car salesman on the lot at a car dealership specializing in expensive, high-end vehicles called out with a megaphone, "Free set of wheels to the next person who buys a car from me!" He was surrounded by stacks of tires and a display case with informational pamphlets advertising all-weather tires. A customer believed that the car salesman was offering a two-for-one deal on the expensive vehicles sold on the lot. Therefore, the customer immediately approached the salesman and purchased a new vehicle for his son, believing that he would have a nice new car to give to his daughter as well. The salesman then asked if the customer would like his free set of tires installed on the new car. The customer responded that he did not want the tires and that the salesman was contractually obligated to give him a second car. The salesman laughed and said that he was never offering a free car but would cancel the sale of the first car as a consolation. Is the salesman obligated to give the customer a free car?

You Selected: No, because no reasonable person would believe that the salesman was offering a free car. No, because the salesman was willing to cancel the sale. Yes, because only the customer has the right to avoid or reform the contract for misrepresentation. Yes, because the customer honestly believed that the salesman was making a valid offer. Under the objective theory of contracts, a party's intent to contract is judged by outward objective facts as interpreted by a reasonable person—not a party's subjective intent or belief. No meeting of minds here.

A charity, seeking to raise funds, held a legally permitted raffle in which the prize was a new automobile. A week before the raffle, the organizer of the raffle contacted a friend who had purchased a raffle ticket. The organizer promised to ensure that the friend would win the raffle if the friend gave the organizer $1,000. The friend agreed and gave the organizer $1,000. On the day before the raffle, the friend began to feel guilty. He went to the organizer, renounced the scheme, and demanded his $1,000 back. The organizer refused. The next day at the raffle, the automobile was awarded to someone else. The applicable jurisdiction makes it a crime to fraudulently conduct a contest, lottery, or prize drawing. If the friend sues the organizer for the $1,000, will the friend be likely to prevail?

You Selected: No, because the agreement between the friend and the organizer was illegal. No, because the friend failed to take any action to prevent the raffle from being held. Correct Answer: Yes, because the friend is entitled to a return of the $1,000 paid to the organizer. Yes, because there was a valid contract between the organizer and the friend. A party to an illegal contract may recover restitution damages if that party conferred a benefit on the other party and (1) was justifiably ignorant of the facts that made the contract illegal, (2) was less culpable than the other party, or (3) withdrew before the contract's illegal purpose was achieved and did not engage in serious misconduct.

A deliveryman for a local bakery was making a delivery when the delivery truck's brakes failed, causing the deliveryman to lose control of the truck. In the resulting collision, the deliveryman sustained substantial injury. The deliveryman informed the bakery's manager that he would be forced to sue the bakery unless the bakery would pay his medical bills relating to the injury as they became due. The manager told him that the bakery would refuse, and threatened to wrongfully fire the deliveryman unless he accepted a payment from the bakery in exchange for signing a document that released the bakery from liability for injuries suffered by the deliveryman. In fear of losing his job, the deliveryman agreed. Subsequently, the deliveryman, upon learning that his injuries from the accident were far more extensive than he had first thought, sued the bakery for negligently failing to maintain a working delivery truck. The bakery moved to dismiss the suit on the basis of the release. The deliveryman, offering to return the payment received from the bakery, moved to set aside the release on the grounds of duress. Should the court grant the deliveryman's motion?

You Selected: Yes, because the bakery's threat deprived the deliveryman of any meaningful choice. Yes, because a contract entered into under duress is void. No, because the bakery itself was not the source of the duress. No, because the deliveryman also threatened the bakery.

A husband contracted with an artist to paint a portrait of his wife using a photograph of her. The husband paid for the portrait in advance, and he wanted the artist to deliver the finished portrait to his wife at her office on their wedding anniversary. The artist agreed to these terms and began painting the portrait. A week later, the husband caught the wife having an affair. Enraged, the husband cancelled his contract with the artist by e-mail. The next day, the wife heard the husband on the phone with the artist discussing what to do with the finished portrait. She heard the husband explain that he no longer wanted the portrait delivered to his wife, and that the artist was free to keep it for his portfolio as long as he returned the husband's money. The wife, wanting to receive the portrait, filed a lawsuit against the artist to enforce the original terms of the contract, and to demand delivery of the portrait.Is the wife's action likely to succeed?

You Selected: No, because the wife's right under the original contract did not vest. No, because the wife was only a donee beneficiary of the contract. Yes, because an intended beneficiary may sue the promisor to enforce a contract. Yes, because the wife did not agree to the modification or rescission of the contract. until the beneficiary's rights vest, the parties to the contract can freely modify or rescind the contract. Because the wife's rights did not vest until after the contract was modified to remove her interest, she cannot sue the artist to enforce the contract.

A homeowner visited the websites of numerous local plumbers for service and hourly rate information. The homeowner called one of the plumbers, told him she had found him through his website, and asked him to unclog a floor drain in her basement. The plumber responded, "Okay, I'll be there tomorrow at three." The plumber unclogged the drain and gave the homeowner an invoice that reflected the hourly rate for his services as posted on his website. The homeowner refused to pay the full invoice amount, asserting that his hourly rate was higher than that charged by other local plumbers. The homeowner offered to pay him the average of those rates. The plumber demanded payment of the invoiced amount. Which of the following describes the compensation the plumber is entitled to receive from the homeowner?

• A. Compensation based on the reasonable value of the plumber's services, because in the absence of an agreement on price, a reasonable price will be supplied. • B. Compensation at the invoiced price, because the hourly rate posted on the plumber's website supplied the price term for the parties' agreement. • C. Compensation necessary to protect the plumber's reliance interest, because he rendered services based on the homeowner's implied promise to pay. • D. Compensation necessary to protect the plumber's restitutionary interest, because there was no meeting of the minds between the parties on price. (Choice A) A court may fix a reasonable price term when the parties fail to do so if the parties intended to enter the contract despite the absence of that term. However, there is no need for the court to do so here since the plumber's website provided the contract's price term.

The defendant, a minor both in fact and appearance, bought on credit and took delivery of a telescope from the 30-year-old plaintiff for an agreed price of $100. Upon reaching her majority soon thereafter, the defendant encountered the plaintiff and said, "I am sorry for not having paid you that $100 for the telescope when the money was due, but I found out it was only worth $75. So I now promise to pay you $75. The defendant subsequently repudiated this promise and refused to pay the plaintiff anything. In an action for breach of contract by the plaintiff against the defendant, the plaintiff's probable recovery is (A) nothing, because the defendant was a minor at the time of the original transaction. (B) nothing, because there was no consideration for the promise made by the plaintiff after reaching majority. (C) $75. (D) $100.

(C) is the best response, because the defendant's partially reaffirmed promise was supported by consideration. Since the defendant was a minor when she bought the telescope in return for her promise to pay $100, her promise was voidable under the rules governing contracts by minors. But when the defendant partially reaffirmed her promise after reaching majority, she fell within the rule that when a person promises to perform all or part of a previous contractual obligation that is voidable at the promisor's option, this new promise is binding even though it is without new consideration. Therefore, the defendant became obligated to pay the newly-promised amount, $75.

At the beginning of the week, a homeowner met with a contractor to discuss remodeling a bathroom in her home. At the conclusion of their meeting, the contractor told the homeowner that he would charge her $5,000 to $6,000 for the work, but that he would get back to her with a final price. When he arrived at his office later that day, the contractor opened an email from the homeowner that she had sent earlier. In the email, she stated that she would pay the contractor $5,000 for the job. Two days later, the contractor responded by email that he could not complete the work for less than $5,500. The homeowner replied by email that she couldn't pay $5,500 but that, if the contractor changed his mind about doing the work for $5,000, he could begin work before the end of the week. The contractor received the email but did not respond. The next day, the contractor appeared at the homeowner's house and began remodeling the bathroom. Which of the following statements regarding the relationship between the parties is most accurate?

A contract was formed at a reasonable price. You Selected: A contract was formed at the price of $5,000. A contract was formed at the price of $5,500. No contract was formed. To form a contract, an offer must be accepted before it terminates. An offer will terminate if, for example, the offeree rejects it by clearly conveying that he/she does not intend to accept the offer OR by making a counteroffer. Once the offer has terminated, it cannot be accepted. But the offer can be revived if the offeror conveys that it is still open. This creates a renewed opportunity for acceptance by the offeree. If the offeree accepts the revived offer, then a binding contract is formed.

A man was moving to another state and decided that he wanted to give away some of his belongings. The man knew that his brother had always expressed interest in the man's antique desk. The man called the brother and said, "I'm going to be moving in two weeks. I would like to give you the antique desk as a gift. I'll drop it off at your house on my way out of town." The brother told the man that he was very grateful for the gift and was looking forward to having the desk in his home office. The brother, in reasonable reliance on the man's promise, immediately disposed of his old desk and made room for the antique one. A couple of days later, an appraiser, who was a friend of the man, visited the man's house for dinner. While at his house, the appraiser saw the antique desk and informed the man that it was worth well over $20,000. The man decided to keep the desk and did not drop it off at the brother's house on his way out of town. The brother brought suit against the man to recover the antique desk. If the court finds in favor of the man on these facts, what is the most likely reason?

A promise to make a gift in the future cannot be enforced. The brother did not rely to his detriment on the man's promise. The man's promise was not in writing. You Selected: The man's refusal to give the antique desk did not cause injustice. Under the doctrine of promissory estoppel, a party's promise to make a gift is enforceable if (1) the promisor should reasonably expect the promisee to rely on the promise, (2) the promisee detrimentally relies on the promise, and (3) injustice can be avoided only by enforcement of the promise.

A private port authority contracted with a company that manufactures and operates cranes to assist with loading and unloading containers from ships docked at the port. One of the company's cranes was defectively manufactured. Due to this defect, a container was dropped, injuring an individual below. The individual sued the port authority, alleging negligence. Neither the individual nor the port authority notified the crane company of this lawsuit. The port authority settled its claim with the individual before trial for a reasonable amount. The port authority seeks to recover the cost of the settlement from the crane company under a breach-of-contract action. Is the port authority likely to prevail?

No, because damages for personal injury cannot be recovered in a breach-of-contract action. No, because the port authority settled the lawsuit rather than litigating the matter to a final judgment. Yes, because the crane company is liable for all consequences flowing from its breach of the contract. You Selected: Yes, because the settlement was reasonably foreseeable at the time the contract was formed. Due to its special circumstances as the dock operator, the port authority suffered damages from the individual's negligence suit. It was reasonably foreseeable that a defect in the crane might cause personal injury and that the port authority, as the dock operator, would be sued for that injury. Therefore, the port authority will likely prevail in its breach-of-contract suit to recover the settlement cost.

A jeweler who specialized in engagement rings assisted a man who was trying to pick out the perfect engagement ring. The man was inexperienced with the various cuts of diamonds and types of ring settings. Over the course of a few weeks, the jeweler and the man looked at all of the ring styles and discussed pricing based on the man's budget of $5,000. The man finally settled upon a square-cut diamond with a prong setting that was priced at $5,500. The man initially offered the jeweler $4,500 for the ring. While the man and the jeweler were negotiating the price, the jeweler received a phone call regarding a family emergency. The jeweler told the man that he would email him an offer in the evening, and if they could "meet halfway," the jeweler would sell the ring to the man. The man agreed. That evening, the jeweler and the man received emails from one another at the same time. The jeweler's email contained an offer to sell the ring for $5,000, and the man's email contained an offer to buy the ring for $5,000. Both emails (i) specified the same style of ring that the two parties had discussed earlier that day, (ii) required payment upon receipt of the ring in two weeks, and (iii) were signed with an electronic signature. Based upon their earlier discussions and the jeweler's email offer to sell the ring to him for $5,000, the man did not look for an engagement ring at any other jewelry store. When the man showed up two weeks later to pick up and pay for the ring, the jeweler denied that they had a binding contract and would not sell the ring. If the man sues the jeweler for breach of contract, which of the following most persuasively supports the man's position?

A sale-of-goods contract does not require that an acceptance be a mirror image of the offer. Correct Answer: Both parties conveyed an intent to contract with one another through prior negotiations and the simultaneous emails. Since the jeweler was the only merchant in the transaction, the jeweler is estopped from denying that the parties' correspondence created a binding contract. You Selected: The man detrimentally relied upon the jeweler's offer to "meet halfway" and the email offer to sell the ring to him.

Prior to her death, a celebrity commissioned an artist to paint a portrait of her. The celebrity hired this particular artist because he painted using an old-fashioned and rarely used style that required two months of daily appointments during which the subject would sit for a few hours each day. The contract between the parties specified that this live-model method would be used and that the celebrity would deliver increasing payments throughout the process, with the first payment occurring after two weeks of painting. One week into the process, after the painting had begun, the celebrity died. Her family demanded that the artist continue with the painting, using photographs as a substitute for the daily sessions. Is the artist required to complete a painting of the celebrity?

No, because no payment had yet occurred. You Selected: No, because the celebrity died after only one week. Yes, because the artist can complete the painting by relying on photos of the celebrity. Yes, because the artist had already begun painting the celebrity. A contracting party's duty to perform can be discharged by impracticability. This defense is available when: an unexpected or extraordinary event makes it impracticablefor the party to perform the contract was formed under a basic assumption that the event would not occur and the party seeking discharge was not at fault in causing the event to occur. In a personal-services contract (e.g., contract to paint a portrait), impracticability can arise when the party who is to perform the contract—or a person whose existence is required for that performance—dies or becomes incapacitated.* Here, the celebrity's existence was required for the artist to perform because the artist needed the celebrity to sit for a few hours each day to complete a live-model portrait as specified in the contract. Therefore, the artist's continued performance became impracticable upon the celebrity's death

As part of a divorce settlement, an ex-husband purchased an annuity from an insurance company to be paid to his ex-wife so that she would receive a fixed amount quarterly for the duration of her life. Within a week after the purchase, the ex-wife learned that she had a fatal illness, which had not previously manifested itself but had existed for some time. She died two months later, prior to receiving any payments from the annuity. The ex-husband has filed suit to rescind the annuity contract. Will the ex-husband be likely to prevail?

No, because the annuity contract was a third-party beneficiary contract. Correct Answer: No, because the ex-husband assumed the risk of his ex-wife's death. You Selected: Yes, because the ex-wife's death frustrated the purpose of the annuity. Yes, because the ex-husband and the insurance company made a mutual mistake as to the ex-wife's health.

Upon the completion of an interview, an attorney offered an assistant $3,000 each month to perform administrative tasks for his law firm. The attorney emailed the assistant an offer confirming these terms. The assistant responded via email, stating that she accepted the offer so long as she could work remotely, expressing that the option to work from home was essential to her. The parties had discussed in the interview that the assistant would likely work in the office but that the attorney would be open to her working remotely. The emailed offer did not address this issue. The attorney did not respond that day but went out and bought the assistant a laptop conforming to her preferences and many office supplies that she had requested. The next day, the attorney emailed the assistant to confirm that he might allow her to work remotely after she demonstrated success within an office environment. She emailed the attorney back, stating that she no longer wished to work with him because she preferred to work from home right away. The attorney threatened to sue her for breach of contract. Does a contract exist that binds the attorney and the assistant?

No, because the assistant had a misunderstanding as to the terms of the contract. You Selected: No, because the assistant included additional terms in her response. Yes, because the assistant emailed him back to accept the offer. Yes, because the attorney's offer did not propose that the assistant could work from home. Under the common-law mirror-image rule, an acceptance must match the terms of the offer exactly to be effective. Therefore, an acceptance acts as a counteroffer—i.e., a rejection of the original offer and a new offer—if it is conditioned upon the offeror's assent to different or additional terms.

A math major at a local college agreed to tutor a high school student who was having trouble in his math class. The tutor promised to meet with the student for five hours a week for the remaining two months of the term, and the student's mother agreed to pay the tutor $20 per hour. The tutor and the mother also agreed that, if the student received a B or better in his math class, the mother would pay the tutor a $500 bonus. The parties did not reduce their agreement to writing. The mother paid the tutor weekly for the tutoring. At the end of the term, the student received a B+ in his math class. The mother, not having the funds to pay the tutor's bonus, contacted the tutor and offered her a bike worth $300 in lieu of the bonus payment. The tutor accepted the bike, which the mother delivered to the tutor shortly thereafter. Is the tutor entitled to recover $200 from the mother?

No, because the bike was worth less than $500. You Selected: No, because the tutor accepted the bike in lieu of the bonus. Yes, because the modification was not in writing. Yes, because there was no dispute as to the amount owed.

On March 1, a bookstore emailed a publisher asking the following: "We have received your catalog and are interested in doing business. Do you offer any discounts to retailers?" On March 2, the publisher emailed the bookstore: "Yes, a retailer receives a 40 percent discount on our books, if at least $500 worth of books are ordered." On March 3, the vendor responded: "Attached is an order for $600 worth of books listed in your current catalog." On March 4, the publisher replied: "Order accepted. Please be aware, as noted in the Terms of Service in our catalog, the cost of shipping for orders of less than $1,000 is borne by the buyer." On March 5, the bookstore emailed: "Can't accept additional shipping cost. Please cancel order." The publisher did not reply. On March 6, the bookstore, having reconsidered its options, sent the following email to the manufacturer: "You drive a hard bargain. Please ship the books as ordered on March 3." If the publisher does not send the books, can the publisher be liable for breach of contract?

No, because the bookstore clearly and unequivocally repudiated the contract on March 5. No, because the publisher's March 2 email was not an offer to form a contract. Yes, because both of the parties are merchants. You Selected: Yes, because the bookstore retracted its repudiation on March 6.

A groom left his bride at the altar on the day of their wedding. The bride could not bear to keep any painful reminders of the occasion, so she offered to sell her wedding dress to one of her bridesmaids for $5,000. The bride stated that the offer would remain open for 30 days. The bridesmaid said that she was interested but would have to think about it. A week later, the bridesmaid emailed the bride to ask if the price included the custom-made veil that the bride had worn. The bride did not respond to the bridesmaid's question. Within the 30-day period, the bridesmaid accepted the bride's initial offer of $5,000 for the wedding dress. In response, the bride stated that the bridesmaid could only buy the wedding dress for $6,000. Was a contract formed when the bridesmaid accepted the initial offer of $5,000?

No, because the bride raised the price of the dress to $6,000. No, because the bridesmaid's question acted as a counteroffer and a rejection of the $5,000 offer price. Yes, because the bride was required to keep the initial offer open for the 30-day period. You Selected: Yes, because the bridesmaid's question did not constitute a counteroffer. since the bridesmaid timely accepted the original offer, the bride could not subsequently raise the price.

A buyer at a local market offered to purchase a large mirror from an artist for $1,000. The artist stated that he wanted to wait to see how many people went through the market that day before he decided on whether he would accept the offer. The buyer agreed to wait until the next morning for the artist's decision. The next morning, the buyer returned to the market only to learn that the mirror had been dropped and shattered. The buyer believed that the destruction of the mirror terminated his original offer, but because the frame of the mirror was still in good condition, the buyer decided to buy the frame instead. The buyer wrote a check for $500 and gave it to the artist without further remark. The artist loaded the empty frame into the buyer's vehicle and, believing that he had accepted the buyer's original offer, demanded the remaining $500 the buyer had offered the day before. Is the buyer liable for the remaining $500?

No, because the buyer believed that the original offer had terminated. You Selected: No, because the original offer terminated. Yes, because the artist thought that he had accepted the original offer. Yes, because the original offer was still valid. An offer can be terminated by operation of law—e.g., when the subject matter of the offer is destroyed.

On April 1, a buyer agreed in writing to purchase an antique car from a seller for $20,000. The parties met on April 10, the scheduled date of the sale, at which time the buyer accepted the car and gave the seller a check for $15,000. The buyer, seeking to create an accord and satisfaction, had added the following conspicuous notation on the check: "This check is in full and final satisfaction of my obligation under our April 1 agreement." The seller did not realize that the check was for only $15,000 and that it contained the notation until the seller sought to deposit it at her bank later that day. Needing the money, the seller deposited the check anyway. If the seller sues the buyer for breach of contract seeking damages of $5,000, the difference between the amount paid and the contract price, will the buyer's accord and satisfaction defense likely succeed?

No, because the buyer could not modify the agreement without consideration. You Selected: No, because the buyer did not dispute the initial purchase price of the car. Yes, because the notation on the check formed a substituted contract. Yes, because the seller deposited the check knowing it was offered in full and final satisfaction of the buyer's obligation.

A political candidate for a statewide office contracted with a songwriter to compose a theme song for his campaign for $10,000. The contract provided that, rather than paying the songwriter, the candidate was to pay the money to the songwriter's daughter. The contract also prohibited the assignment of any rights or duties arising under the contract without the permission of the other party. Subsequently, the daughter, upon learning of the contract from her father, decided to donate this money to a charity and transferred her rights under the contract to the charity. She did not secure the permission of the political candidate before making this transfer. The songwriter composed the song, which the candidate used in her campaign, but the candidate did not pay anyone for the song. The charity filed suit against the candidate for breach of contract. The charity has taken no action and has incurred no expenses related to the contract.Will the charity prevail in its action against the candidate?

No, because the charity has not detrimentally relied on the candidate's promise. You Selected: No, because the contract prohibits assignments. Correct Answer: Yes, because the charity is the assignee of the daughter's contractual rights. Yes, because the charity is the donee beneficiary of the contract. a prohibition against the assignment of a contract does not prevent an assignment of the right to receive payments due, unless circumstances indicate the contrary.

A party-planning company specialized in creating and selling nine different kits for themed parties. A store that sells party-related items entered into a written agreement with the company. Under this agreement, the company was to deliver 500 kits to the store by November 1. The agreement stated that selections regarding the types of kits and the number of each were to be made by October 15, but the agreement did not specify who was to make the selections. Neither the store nor the company selected any assortment of the kits by October 15. On October 16, the company notified the store that due to its breach, the company would not be shipping the party kits. On October 17, after receiving the company's notification, the store informed the company of its selections. The company refused to send the kits that the store selected even though it had a surplus of all of the merchandise and could have filled the store's order with any combination of themed kits. If the store sues the company for breach of contract on November 2, is the store likely to prevail?

No, because the company had no duty to perform since an assortment was not selected by October 15. No, because the failure to specify the party responsible for selecting the types and numbers of each kit renders the contract unenforceable due to the indefiniteness of its terms. Yes, because the company was required to make a reasonable selection of available merchandise to fill the order. You Selected: Yes, because the store's two-day delay in making its selections did not have a material effect on the company's ability to perform the contract.

A tenant rented a small cabin from a landlord. The lease provided that the tenant was permitted to make structural improvements to the cabin but that the tenant must pay for such improvements. Relying on this clause in the contract, the tenant contacted a contractor to install a loft in the cabin for $10,000. The tenant and the contractor agreed in a writing signed by both parties that payment would be due 30 days after the loft was completed. The contractor knew that the tenant was renting the cabin and sent the landlord a letter informing him of the impending construction on his property. The landlord received the letter and did not reply. The contractor completed the loft, which increased the market value of the cabin by $6,000. Ten days later and three months before the end of her lease, the tenant vacated the cabin and disappeared. Thirty days after the loft was completed, the contractor's bill remained unpaid. If the contractor has no remedy quasi in rem under the jurisdiction's mechanic's lien statute, which of the following will give the contractor the best chance of recovery in personam against the landlord?

An action as a third-party beneficiary to the lease. An action based on an implied-in-fact contract. An action based on promissory estoppel. You Selected: An action in quasi-contract for the benefit conferred on the landlord. A "quasi in rem" remedy is effective against a defendant in reference to property, while an "in personam" remedy is effective against a defendant directly. In personam remedies are available for most causes of action, including quasi-contract. A plaintiff can recover in quasi-contract—despite having no contractual relationship with the defendant—if the plaintiff conferred a non-gratuitous and measurable benefit on the defendant that resulted in unjust enrichment because: the defendant had the opportunity to decline the benefit but knowingly accepted it or the plaintiff had a reasonable excuse for not giving the defendant an opportunity to decline.

At an auction without reserve, the auctioneer called for bids for an antique chair. The first bidder, a consumer without specialized knowledge about antique furniture, bid $10,000. Her bid was acknowledged by the auctioneer. The second bidder bid $11,000, which was also acknowledged by the auctioneer. Before the auctioneer announced the sale of the item to the second bidder, she withdrew her bid. The auctioneer then announced that the chair was sold to the first bidder for $10,000. Can the first bidder successfully challenge this sale?

Answer Choices:Yes, because the first bidder was not a merchant.Yes, because the withdrawal of the highest bid did not reinstate the next highest bid.No, because the auctioneer accepted the first bidder's bid.No, because the auction was without reserve. At a reserve or no-reserve auction, a bidder has the right to withdraw a bid until the auctioneer announces the completion of the auction sale. However, the bidder's retraction will not revive any prior bids.

In need of money, the owner of a ring prepared an email one evening proposing to sell the ring to a friend for $500, but only if he responded within 24 hours. Unable to bring herself to send the email, the owner, who normally was a teetotaler, began drinking. When she was thoroughly intoxicated, she sent the email without realizing it. After the owner sobered up the following afternoon, she called her friend and said that she had never meant to send the email, but her friend informed her that he had already responded by email, agreeing to the transaction. Does a valid contract exist?

Answer Choices:Yes, because the friend accepted the owner's offer to sell the ring.Yes, because the friend had 24 hours in which to respond.No, because the owner lacked capacity at the time that she made the offer.No, because the contract was executory. Learner Selected Answer: Yes, because the friend accepted the owner's offer to sell the ring. Here, the owner emailed the friend and offered to sell him the ring for $500 if he responded within 24 hours. The offer had to be accepted by that specified time or it would lapse. The friend accepted the offer before it lapsed by emailing the owner and agreeing to the transaction. And since the owner had not revoked the offer before it was accepted, a valid contract exists.

A wholesaler of bicycle chains sent a retailer the following fax on December 1: "Because of your continued loyalty as a customer, I am prepared to sell you up to 1,000 units of Bicycle Chain Model D at $7.50 per unit, a 25% discount off our original $10.00 price. This offer will remain open for 7 days." The fax lacked a full, handwritten signature, but it was on the wholesaler's letterhead and had been initialed by the wholesaler's head of sales. On December 4, the wholesaler's head of sales called the retailer and informed the retailer that the wholesaler had decided to revoke his December 1 offer. On December 5, the retailer placed an order for 1,000 bicycle chains, stating that he would pay the discounted price of $7.50 per unit. What is the correct value of the order placed by the retailer?

Answers: $7,500, because the wholesaler's revocation was not in writing. Correct Answer: $7,500, because the wholesaler was bound to keep the offer open for 7 days. You Selected: $10,000, because the offer was not signed by the wholesaler. $10,000, because the retailer did not provide consideration to hold the offer open. Contracts for the sale of goods are governed by the UCC. Under the UCC firm-offer rule, a merchant's signed, written offer to buy or sell goods may contain assurances that it will remain open. If so, the offer is irrevocable for the time stated in the offer or, if no time is stated, for a reasonable time. The signed offer need not always contain a full, handwritten signature, such as when: initialing the relevant clause is appropriate under the circumstances or the merchant handwrites on his/her letterhead to confirm that a firm offer was made.

On May 10, the coach of a youth-league baseball team sent a letter to a supplier asking the supplier to promptly ship 20 red jerseys to the coach. On May 15, the supplier received this letter and sent the coach a reply letter accepting the offer. On May 16, the supplier realized that he had no red jerseys with which to fill the order and sent the coach 20 blue jerseys with a note that the blue jerseys were tendered as an accommodation. The coach received the jerseys and accommodation note on May 18 and received the supplier's acceptance letter on May 19. On May 20, which of the following is a correct statement of the parties' legal rights and duties?

Answers: Correct Answer: The coach can either accept or reject the blue jerseys and, in either event, recover damages, if any, for breach of contract. The coach can either accept or reject the blue jerseys, but if he rejects them, he will thereby waive any remedy for breach of contract. The supplier's shipment of nonconforming goods constituted an acceptance of the coach's offer, thereby creating a contract for the sale of the blue jerseys. You Selected: The supplier's shipment of the blue jerseys constituted a counteroffer. Contracts for the sale of goods (here, jerseys) are governed by Article 2 of the Uniform Commercial Code (UCC). Under the UCC, an offer to buy goods for prompt shipmentcan be accepted by: promising to ship the goods shipping goods that conform to the order or shipping nonconforming goods without notice that they are being offered only as an accommodation (which also operates as a breach)

A sister convinced her brother that they should open a small coffee shop. Their friend, a guitarist, suggested bringing his band to play live music in order to attract customers. He did not request any payment, saying that the publicity would be good for the band. The siblings agreed, and the band started playing at the coffee shop weekly. The coffee shop became a success, in no small part due to the band's performances. When a businessperson offered to buy the coffee shop from the siblings, they orally agreed to each pay the guitarist $10,000 out of their share of the sale proceeds for his help in making the shop popular. The sister told the guitarist about their agreement. He was so delighted with it that he made a down payment on a new car. By the time the sale of the coffee shop was finalized, the brother had encountered financial difficulties. After the sale, the siblings signed a written contract stating that the sister would pay the guitarist $10,000 and her brother would pay him $5,000. If, after the sale, the brother pays the guitarist only $5,000, will the guitarist have a valid basis for an action against the brother for another $5,000?

Answers: No, because the guitarist was bound by the written modification of the contract made by the siblings. You Selected: No, because the guitarist was only a donee beneficiary of the oral contract between the siblings. Correct Answer: Yes, because the guitarist's reliance on the promised payment prevented the siblings from changing the obligations of their oral contract. Yes, because the oral promise to pay $10,000 to the guitarist was made binding by the guitarist's valuable and uncompensated contributions to the business.

A student inherited a large tract of undeveloped land from an eccentric uncle. The student had no present need for the land, and because he had numerous student loans, he decided to sell the land. He advertised aproposed sale of the property, and he was soon contacted by a rancher who owned property adjacent to the offered land. The rancher wanted to purchase the student's property to expand his ranch and to build facilities for dairy production. The student told the rancher that his car had just broken down and that he was eager to sell the property quickly so that he could repair his car for his commute to class. Although the rancher was fully aware of the fair market value of the property, he offered the student a cash price 80 percent less than the property was worth. The student, disappointed with the low price but desperate to repair his car, accepted the rancher's offer. On these facts, which of the following legal concepts would give the student the best chance of canceling the contract with the rancher?

Bad faith. Duress. Equitable estoppel. You Selected: Unconscionability. A duty of good faith and fair dealing is imposed on each party in the performance and enforcement of an existing contract—not in contract formation. While acting in bad faith during contract negotiations may have consequences, bad faith alone is not a defense to contract formation.

An independent trucker and a manufacturer entered a written contract for the delivery of a farming implement from the manufacturer to a farmer. Under the terms of the contract, the trucker promised "to deliver a farming implement from the manufacturer to the farmer," and in exchange, the manufacturer promised "to pay the trucker if the trucker delivers the implement directly to the farmer after picking it up." The trucker picked up the implement but, instead of driving directly to the farmer, drove 100 miles out of his way to pick up another item from a third party before delivering the implement to the farmer. The manufacturer, unaware that the trucker had failed to deliver the implement directly to the farmer, refused to pay the trucker. Who has breached this contract?

Both the trucker and the manufacturer. You Selected: The trucker only. The manufacturer only. Correct Answer: Neither the trucker nor the manufacturer. If the parties expressly agree to a condition precedent, then the condition precedent will be strictly enforced. This means that a contracting party must fully comply with the condition before the other party's performance is due. (Objective standard, regardless of whether the party knew the other side failed to perform condition precedent).

Prior to the beginning of the season for a recreational softball league, the manager of one of the teams phoned a friend who has a shop that imprints and sells various types of clothing. The manager told the friend that he needed 20 customized jerseys in the team's color, green, with the team's name imprinted on the front and each player's name on theback. The friend agreed to do so at the cost of $30 per jersey. The friend asked the manager to send her an e-mail, as a reminder of their agreement. The manager sent a signed e-mail the following day that contained the terms of the agreement, except that the manager mistakenly typed "17" rather than "20" as the number of jerseys needed. The friend imprinted 20 green jerseys with the team's name and each player's name, and called the manager to tell him to come pick up the jerseys. The manager informed the friend that he no longer needed the jerseys because the league had disbanded due to an inability to field enough teams. When the manager refused to pay for the jerseys, the friend sued him for $600 in damages. The manager asserted the Statute of Frauds as a defense.What amount of damages is the court likely to award the friend?

Correct Answer: $600, because the jerseys were specially manufactured goods. $600, because the written agreement may be reformed to reflect the parties' agreed-upon quantity. You Selected: $510, because the contract is enforceable only to the quantity stated in the manager's signed e-mail. $0, because there is no writing signed by the manager that reflects the agreed-upon quantity. specially manufactured goods are exempted from SOF.

An amateur bodybuilder entered into a contract with a personal trainer to help the bodybuilder prepare for his first bodybuilding competition. The written agreement stated that the bodybuilder would pay the personal trainer $4,000 to do daily weightlifting sessions and teach him about competition musculature and proper nutrition. Per the contract, the training would occur over a period of eight weeks leading up to the competition, and the bodybuilder would pay the personal trainer at the end of the eight weeks. Three weeks before the competition, the bodybuilder was lifting weights with the guidance of the personal trainer when a beam suddenly fell from the gym's ceiling and onto the bodybuilder. The bodybuilder suffered severe head trauma and was put into a medically induced coma for more than a month. With regard to the contractual duties of each party, which of the following is a correct statement?

Each party's duty to perform is dismissed, and the personal trainer cannot recover anything from the bodybuilder. You Selected: Each party's duty to perform is dismissed, but the personal trainer can recover restitution damages for the weeks that he did train the bodybuilder. Only the personal trainer's duty to perform is dismissed, but he can recover reliance damages for the weeks that he did train the bodybuilder. Only the personal trainer's duty to perform is dismissed, but the bodybuilder is still obligated under the contract because the amount that he owed the personal trainer can be determined. Performance is discharged by impracticability when (1) an unforeseen event occurs, (2) nonoccurrence of the event was a basic assumption of the contract, and (3) the party seeking discharge did not cause the event. However, restitution damages are recoverable for benefits conferred before performance was discharged. When, as here, performance is discharged by impracticability, both parties are no longer required to perform

A farmer owned a tractor and offered his brother the chance to purchase it. The farmer told the brother that he had to decide whether he wanted to purchase the tractor within "six months of today's date." The brother paid the farmer $200 that day to keep the option open. The agreement was reduced to writing, signed by both men, and dated May 15. The farmer died on July 1. On August 15, the brother notified the executor of the farmer's estate that he wanted to accept the offer to buy the tractor. The executor refused to sell, and the brother filed suit for the enforcement of the contract. Is the brother likely to prevail?

No, because at the time of the farmer's death, the tractor went to his estate. You Selected: No, because the offer terminated on July 1. Yes, because the brother made an enforceable contract to buy the tractor on May 15. Correct Answer: Yes, because the brother paid $200 to keep the option open. An offer to form a contract generally remains open for the time stated in the offer or, if no time is stated, for a reasonable time. However, an offer will terminate prematurely if the offeror dies or becomes mentally incapacitated before the offer is accepted—with one exception. An option contract will not terminate under such circumstances because the offeree gave separate consideration to keep the offer open for a specified period of time.* This offer was irrevocable for the entire option period (six months) even though the farmer died on July 1.

A mechanic and a farmer contracted in writing for the repair of the farmer's tractor, with a payment of $2,000 due upon completion. The mechanic called the farmer on April 15 to inform him that the work was complete. When the farmer went to pick up the tractor the next day, he told the mechanic that due to an unforeseen rise in feed costs, he could not pay the full contract price. The farmer paid the mechanic $1,000. The mechanic told the farmer that, if the farmer promised to pay the remainder by June 1, then the mechanic would not sue to recover the remaining $1,000. The farmer orally agreed. On May 1, the mechanic sued the farmer for the unpaid $1,000, and the farmer filed a motion to dismiss. Should the court grant the motion to dismiss?

No, because the new cost of feed is not an unforeseen difficulty that would allow for modification of the existing contract. You Selected: No, because there is no consideration to support the mechanic's promise not to sue. Yes, because a promise to allow a debtor to delay payment on a past debt is enforceable without consideration. Yes, because the payment of $1,000 was sufficient consideration to support the mechanic's promise not to sue. Under the preexisting-duty rule, a promise to perform a duty that a party is already legally bound to perform is not consideration.

On April 1, a buyer and a seller executed a written contract for the sale of an antique car for $40,000, delivery on May 1. The contract contains a clause indicating that it is a total integration of the parties' agreement. As they each signed the contract, the buyer orally reminded the seller that the buyer's duty to purchase the car was conditioned on his ability to get approval for a loan by April 20 to fund the purchase. The seller orally agreed, though the condition was not noted in the written contract. When the seller contacted the buyer to execute the sale on May 1, he discovered that the buyer had attempted but failed to get a loan and could not afford to purchase the car. The buyer refused to honor the contract. If the seller sues the buyer for breach of contract, will the court likely admit the evidence of the oral condition regarding the buyer's approval for a loan?

No, because the oral agreement contradicts the terms of the written contract. No, because the written contract is a complete integration of the agreement between the parties. You Selected: Yes, as proof of a condition precedent to the buyer's obligation under the contract. Yes, because the oral agreement was a distinct and separate contract.

A nonprofit organization that provides summer camps for at-risk youth hosted a walkathon to raise money to provide stipends for summer-camp expenses to 350 low-income families. The organization solicited various businesses to sponsor the walkathon, including an executive of a large food-services company. The executive wanted the organization to hire his food-services company to provide food services to the organization's summer camps when the organization's contract with their current food-services provider expired in one month. Hoping to catch the organization's attention, the executive pledged that his company would match all of the sponsors' pledges. Because he was a business sponsor, the executive was required to fill out and sign a writing stating this promise. The walkathon was a success, and the total amount raised, excluding the executive's pledge, was $42,000. After collecting the pledge money from all of the sponsors except for the executive, the organization was able to issue stipends to all 350 families. The organization planned to use the executive's pledge to repair some of the older cabins at the summer camp. Prior to being asked for his matching pledge, the executive learned that the contract of the organization's current food-services provider had been renewed for another five years. The executive subsequently repudiated his promise to match all of the sponsors' pledges. If the nonprofit organization sues the executive for $42,000, will it likely succeed?

No, because the organization did not actually rely to its detriment on the executive's promise to match all of the sponsors' pledges. You Selected: No, because there is no substantial injustice since the organization was able to issue all 350 stipends without the executive's pledge. Yes, because the executive acted in bad faith when he repudiated his promise to match the pledges of the other sponsors. Correct Answer: Yes, because the executive's promise to match the pledges of the other sponsors is enforceable without proof of detrimental reliance or substantial injustice.

Due to recent financial difficulties, a man asked his brother if he would be willing to loan him $3,000. The brother agreed to make the $3,000 loan to the man. Under the terms of their agreement entered into onDecember 31, the man would be responsible for making monthly payments of $125 plus interest for the next two years at the beginning of each month, starting January 1. Pursuant to the agreement with his brother, the man made payments each month for the first six months. However, he failed to make the agreed-upon payments for July and August. On August 30, the man told his brother that he could no longer afford to repay him. The brother filed suit against the man on August 31 for breach of contract. What amount, if any, is the brother able to recover on August 31?

Nothing, because the agreement did not contain an acceleration clause. Correct Answer: $250 plus interest, the amount owed for July and August. $375 plus interest, the amount owed for July, August, and September. You Selected: $2,250 plus interest, the amount outstanding on the loan, because the man repudiated the contract. An installment contract is breached when a party fails to perform any installment. A breach by nonperformance accompanied by a repudiation generally constitutes a total breach. But it constitutes a partial breach when the only remaining duty at the time of the breach (1) is held by the breaching party and (2) is for the payment of money in unrelated installments. Here, The brother can recover for the two missed payments in July and August—but not the amount not due yet.

A borrower failed to make several payments due on a business loan. While the borrower was in the hospital recovering from a major operation, the lender, who was a family friend, approached the borrower's wife about additional security for the business loan. The lender intimated that, if some action was not taken with regard to the loan, the lender would have to file a civil action. Distraught over her husband's physical condition, the borrower's wife, at the lender's suggestion, granted the lender a mortgage on valuable property that she had recently inherited. The mortgaged property was to serve as additional security for the business loan. As a consequence of the mortgage, the lender did not reduce the outstanding amount of the business loan but did extend the time in which the borrower had to repay the loan. Of the following, which would be the best basis on which the borrower's wife can seek to avoid the mortgage?

The borrower's wife granted the mortgage under duress. The borrower's wife lacked the capacity to enter into a contract. The lender did not give additional consideration for the mortgage. You Selected: The mortgage was obtained through undue influence.

The owner of a retail clothing store regularly displayed for-sale works by local artists on a wall in the store. An art collector who came into the store inquired about purchasing a particular work for display at his home. The two agreed upon a price, but the collector was not ready to commit to purchasing it immediately. Confident that the collector would purchase the work, the owner promised in a signed writing to sell the work to the collector at the agreed-upon price at any time before the end of the month. On the last day of the month, the collector sent the owner a check for the agreed-upon price, which the owner received on the following day. If the owner returns the collector's check and refuses to sell the artwork to the collector, which of the following best supports the owner's position that a contract had not been formed?

The collector could not accept the owner's offer by mailing a check. Correct Answer: The collector's acceptance of the owner's offer was not timely. The firm-offer rule is not applicable because the collector was not a merchant with respect to the artwork. You Selected: The firm-offer rule is not applicable because the owner was not a merchant with respect to the artwork. Mailbox rule does not apply to firm offers, options, or other irrevocable offers. Acceptance of a firm or otherwise irrevocable offer is effective only if it is received by the offeror before the offer expires.

An honest dispute developed between a condominium owner and a plumber over whether plumbing installed in the kitchen and bathrooms of the condominium satisfied contractual specifications. If the plumbing met those specifications, the condominium owner would owe the plumber $15,000 under the terms of the contract. The condominium owner offered to pay the plumber $10,000 in satisfaction of the owner's contractual obligations if the plumber replaced the plumbing in the kitchen with another grade of pipe. The plumber accepted the condominium owner's offer. After the plumber replaced the kitchen plumbing, the condominium owner refused to pay the plumber. In a breach-of-contract action brought by the plumber, the fact finder determined that the plumbing originally installed by the plumber did satisfy the contract specifications. The fact finder also determined that the plumber and the condominium owner entered into a substitute agreement under which the owner failed to deliver the required performance. What is the maximum amount that the plumber can recover in damages from the condominium owner?

$25,000. You Selected: $15,000. Correct Answer: $10,000. Nothing. Accord agreement - when a party agrees to accept different performance in satisfaction of (i.e., in place of) the original promise; after breach, the party can sue under either the original contract or the accord agreement. Substitute contract - when the parties form a second agreement that immediately discharges the original contract; after breach, a party can sue under the substitute contract only. The more formal the agreement (e.g., words discharging original duties, consideration on both sides), the more likely the fact finder will determine that the parties intended to create a substitute contract.

A carpenter owed a bank $22,000 on an outstanding personal loan. The carpenter and a homeowner entered into a contract under which the carpenter agreed to perform work for the homeowner and the homeowner agreed to pay $22,000 to the bank within 30 days of completion of the work. The carpenter completed the work, but the homeowner incurred costs of $2,000 to correct minor deficiencies in the work. What amount, if any, is the homeowner obligated to pay the bank? (A) $20,000, because the bank, as an intended beneficiary of the homeowner's contract with the carpenter, stands in the position of the carpenter. (B) $22,000, and the homeowner can seek damages from the carpenter for the deficiencies in the work. (C) Nothing, because the bank is an incidental beneficiary of the homeowner's contract with the carpenter. (D) Nothing, because the carpenter's breach discharged the homeowner's obligation to pay the bank.

(A) is the best response, because the bank, as an intended beneficiary of the carpenter-homeowner agreement, can recover from 249 the homeowner for breach of that agreement, but the homeowner can offset this recovery by its claim for the carpenter's partial breach. This is a third-party-beneficiary ("TPB") situation. The bank is an "intended beneficiary" of the contract between the carpenter and the homeowner - that is, the original parties intended the contract to benefit that beneficiary. An intended beneficiary can enforce a contract to which it is not a party, Rest. 2d, Contracts, § 302. This means that here, the intended beneficiary (the bank) can recover the $22,000 that the homeowner promised to pay the carpenter. However, in all TPB situations, the defendant (here, the homeowner) can assert, against the plaintiff-beneficiary (the bank), any claims the defendant would have for breach if the defendant had been sued by the defendant's original counter-party (the carpenter) rather than by the beneficiary. This means that the homeowner, as recipient (promise) of the carpenter's promise to perform the work, can assert against the bank as an offset the homeowner's $2,000 claim for partial breach of the carpenter's promise to the homeowner. Rest. 2d, Contracts, § 309(2).

On December 15, a lawyer received from a stationer an offer consisting of its catalog and a signed letter stating, "We will supply you with as many of the items in the enclosed catalog as you order during the next calendar year. We assure you that this offer and the prices in the catalog will remain firm throughout the coming year." No other correspondence passed between the stationer and the lawyer until the following April 15 (four months later), when the stationer received from the lawyer a faxed order for "100 reams of your paper, catalog item #101." Did the lawyer's April 15 fax constitute an effective acceptance of the stationer's offer at the prices specified in the catalog? (A) Yes, because the stationer had not revoked its offer before April 15. (B) Yes, because a one-year option contract had been created by the stationer's offer. (C) No, because under applicable law the irrevocability of the stationer's offer was limited to a period of three months. (D) No, because the lawyer did not accept the stationer's offer within a reasonable time.

(A) is the best response, because, although the offer was no longer irrevocable on April 15, it had not been revoked, and was therefore capable of being accepted. UCC § 2-205 allows merchants to make "firm offers, i.e., offers that are irrevocable for a limited period even without consideration. Section 2-205 says that the period of irrevocability "in no event may... exceed three months." So by April 15, the offer was no longer irrevocable, and the stationer was therefore free to revoke it. But the stationer did not in fact revoke. (The revocation could only have occurred if the lawyer had learned that the stationer was no longer sticking to the offer.) Nor did the offer lapse on account of the passage of time, because the circumstances indicate that a "reasonable time for acceptance" was any time during the following year. There is no other event that caused the offer to terminate. Consequently, the lawyer's fax was a valid acceptance.

A computer retail outlet contracted to service a bank's computer equipment for one year at a fixed monthly fee under a contract that was silent as to assignment or delegation by either party. Three months later, the retail outlet sold the service portion of its business to an experienced and well-financed computer service company. The only provision in the agreement between the retail outlet and the computer service company relating to the outlet's contract with the bank stated that the outlet "hereby assigns all of its computer service contracts to the computer service company." The computer service company performed the monthly maintenance required under the service contract. Its performance was defective, however, and caused damage to the bank's operations. Whom can the bank sue for damages arising from the computer service company's defective performance? (A) The retail outlet only, because the computer service company made no promises to the bank. (B) Either the retail outlet or the computer service company, because the bank has not released the outlet and the bank is an intended beneficiary of the outlet's agreement with the computer service company. (C) Either the retail outlet or

(B) is the best response, because a delegation does not release the delegator, and the delegate is deemed to have promised to perform the delegated services. First, let's look at whether the retail outlet may be sued. The outlet effectively delegated to the computer service company the outlet's duty to perform for the bank. This happened automatically when the outlet said that it "assigns all of its computer service contracts[.] " That's because an assignment "of the contract" automatically includes a delegation of duties. See Rest. 2d § 328(1): "Unless the language or the circumstances indicate the contrary ... an assignment of 'the contract' or of 'all my rights under the contract' or an assignment in similar general terms is an assignment of the assignor's rights and a delegation of his unperformed duties under the contract." But a delegation of duties does not, unless the obligee expressly agrees, release the assignor/delegator. See Rest. 2d § 318(3): "Unless the obligee agrees otherwise, neither delegation of performance nor a contract to assume the duty made with the obligor by the person delegated discharges any duty or liability of the delegating obligor." So even though the outlet successfully delegated its duties, and the service company assumed them, the outlet remained liable to the bank, because the bank did not specifically agree to release the outlet from these liabilities. Now, let's look at whether the service company is liable. When a contracting party assigns "the contract to an assignee, and the assignee accepts the assignment, the assignee is deemed to have promised to perform the assignor's duties, and the obligee is an intended beneficiary of that implied promise. Rest. 2d § 328(2). So when the outlet assigned all of its service contracts, and the service company accepted the assignment, the service company was deemed to have made a promise to perform the outlet's not-yet-performed services under those contracts, and the bank was deemed to be an intended beneficiary of that promise of performance by the service company. Since an intended beneficiary of a promise may sue the promisor for breach, the bank may recover against the service company.

A wholesaler contracted in a signed writing to sell to a bakery 10,000 pounds of flour each week for ten weeks, the flour to be delivered to the bakery on Mondays and payment to be made on Wednesdays of each week. The bakery did all of its weekly bread baking on Tuesdays. On Monday morning of the first week, the wholesaler tendered delivery of 8,000 pounds of flour to the bakery, and the bakery accepted it on the wholesaler's assurance that the remaining 2,000 pounds would be delivered later that evening, which it was. The bakery paid for both deliveries on Wednesday. On Monday of the second week, the wholesaler tendered delivery of 5,000 pounds of flour to the bakery and said that the remaining 5,000 pounds could not be delivered on Monday but would be delivered by Wednesday. The bakery rejected the tender. Was the bakery legally justified in rejecting the tender of the 5,000 pounds of flour? (A) Yes, because the bakery was legally entitled to reject any tender that did not conform perfectly to the contract. (B) Yes, because the tender was a substantial impairment of that installment and could not be cured. (C) No, because the tender was not a substantial impairment of the entire contra

(B) is the best response, because it correctly captures the rule on when defective installments may be rejected. In sale-of-goods cases, it is actually harder for the buyer under an installment contract to reject a nonconforming tender than for the buyer in a one-shot 264 (non-installment) contract. Rejection in installment contracts is handled by a special code section, S 2612(2), which begins by saying that "the buyer may reject any installment which is non-conforming if the non -conformity substantially impairs the value of that installment and cannot be cured!.! " The final clause of that section then adds a proviso dealing further with cure: If the non-conformity of the installment does not substantially impair the value of the whole contract, then "if ... the seller gives adequate assurance of (the non-conformity's] cure the buyer must accept that installment." Applying § 2-612(2) to these facts: (1) The nonconformity certainly substantially impaired the value of the installment, since 50 percent of the required poundage was missing. (2) The non- conformity couldn't be "cured" here, since a cure would require that the shortfall be made up by the time of the weekly baking on Tuesday morning, and the seller was merely committing to make up the shortfall on Wednesday, which would be too late to be useful. (3) For the same reason as (2), the seller's assurances of cure were not "adequate," since the make-up delivery would arrive too late for the week's baking. So Choice (B) correctly explains why the requirements for the buyer to reject under $2-612(2), first clause, are satisfied.

An engineer signed a two-year contract to serve as the chief safety engineer for a coal mine at a salary of $7,000 per month. The position required the engineer to work underground each workday. After one week on the job, the engineer became very ill. He requested testing of the mine air system, which revealed the presence of a chemical agent to which the engineer had a rare allergic reaction. The engineer promptly quit. The coal mine then hired a qualified replacement for the remainder of the engineer's contract at a salary of $7,500 per month. Assume that no statute or regulation applies. If the coal mine sues the engineer for breach of contract, is the coal mine likely to recover damages? (A) No, because an at-will employee has the right to terminate an employment contract. (B) No, because the risk to the engineer's health excused his nonperformance of the contract. (C) Yes, because the coal mine acted in good faith in mitigating the effect of the engineer's failure to finish the contract term. (D) Yes, because the mine is reasonably safe for most people.

(B) is the best response, because the engineer's allergy made it impracticable for him to perform the contract without undergoing a health risk disproportionate to the benefits to the mine from having that engineer rather than a substitute complete the contract. 351 This question presents a classic illustration of the principle that "supervening impracticability" may act to discharge a party from its duties under a contract. Let's look at how the Second Restatement explains the supervening impracticability principle: § 261 Discharge by Supervening Impracticability Where, after a contract is made, a party's performance is made impracticable without his fault by the occurrence of an event the non-occurrence of which was a basic assumption on which the contract was made, his duty to render that performance is discharged, unless the language or the circumstances indicate the contrary.

In order to raise revenue, a city required home-repair contractors who performed work within the city limits to pay a licensing fee to a city agency. A contractor who was unaware of the fee requirement agreed to perform home repairs for a city resident. After the contractor completed the work, the resident discovered that the contractor had not paid the licensing fee, and she refused to pay for the repairs, which were otherwise satisfactory. If the contractor sues the resident for breach of contract, how is the court likely to rule? (A) Although the contract violates the law and is void, the court will require the homeowner to pay the contractor the reasonable value of the work accepted. (B) Although the contract violates the law, the court will find that public policy does not bar enforcement of the contract, because the purpose of the fee is merely to raise revenue. (C) Because the contract violates the law and is void, the court will not enforce it. (D) Because the purpose of the fee is merely to raise revenue, the court will find that the contract does not violate the law but will allow the contractor to recover his costs only.

(B) is the best response, because where a licensure requirement is designed merely to raise money, and a provider has rendered services without complying with the requirement, a court will not refuse to enforce the agreement on public policy grounds. Contrary to what you may think you remember from law school, there is no general doctrine to the effect that where performance of a contract would violate the law (i.e., the "contract would be illegal"), the court will refuse to enforce that contract. Instead, the only relevant doctrine is that certain agreements will be "unenforceable on grounds of public policy." See, e.g., Rest. 2d, § 178(1): § 178 When a Term Is Unenforceable on Grounds of Public Policy (1) A promise or other term of an agreement is unenforceable on grounds of public policy if legislation provides that it is unenforceable or the interest in its enforcement is clearly outweighed in the circumstances by a public policy against the enforcement of such terms.

A 50-year-old uncle wrote to his adult, unemployed niece and said: 'If you come and live with me and take care of me and my farm for the rest of my life, I will leave the farm to you in my will." The niece immediately moved in with her uncle and took care of him and the farm until the uncle was killed instantly in an automobile accident two weeks later. By his will, the uncle left his entire estate, including the farm, to his unmarried sister. The farm was reasonably worth $75,000. Which of the following best states the rights of the niece and the uncle's estate (or sister)? (A) The niece is entitled to receive the reasonable value of her two weeks services only, because two weeks service would be inadequate consideration for the conveyance of the farm. (B) The niece is entitled to receive the reasonable value of her two weeks services only, because the uncle's letter was an invalid promise to make a will. (C) The niece is entitled to receive a conveyance of the farm, because the letter and her services created a valid contract between her and her uncle. (D) The estate (or sister) has the right to rescind the contract, if any, because the uncle's death within two weeks after the agreemen

(C) is the best response, because it correctly identifies that the niece will be entitled to the farm due to a valid, enforceable contract with her uncle. Under these facts the uncle made his niece an offer, in his letter - that is, a promise to do something in return for something else. He promised to will the farm to his niece in return for her caring for him and the property for the rest of his life. Since he sought performance in return, not a promise, he proposed a unilateral, not a bilateral, contract. By moving in with him and commencing taking care of him and the property, his niece accepted his offer. By remaining at the farm until his death, she fully performed under the contract, thus entitling her to the uncle's return performance: transferring the farm to her. Since (C) identifies this, it's the best response.

In a written contract, an architect agreed to draw up plans for and to supervise construction of a client's new house. In return, the client agreed to pay the architect a fee of $10,000 to be paid upon the house's completion. After completion, the client claimed erroneously but in good faith that the architect's plans were defective. The client orally offered to pay the architect $7,500 in full settlement of the claim for the fee. The architect orally accepted that offer despite the fact that the reasonable value of his services was in fact $10,000. The client paid the architect $7,500 pursuant to their agreement. The architect subsequently sued the client for the remaining $2,500. In a preliminary finding, the trier of fact found that there were no defects in the architect's plans. Will the architect be likely to prevail in his action against the client for $2,500? (A) Yes, because payment of $7,500 cannot furnish consideration for the architect's promise to surrender his claim. (B) Yes, because the oral agreement to modify the written contract is not enforceable. 318 (C) No, because the architect's promise to accept $7,500 became binding when the client made the payment. (D) No, becaus

(C) is the best response, because it correctly states that the architect's acceptance of the $7,500 payment prevents him from any further recovery. The doctrine of accord and satisfaction will prevent the architect from further recovery. An "accord" is an agreement under which a party to a contract agrees to accept, as complete satisfaction of the contract, some performance different from that originally due under the contract. "Satisfaction" is performance of the accord, and once satisfaction takes place, both the accord and the original contractual duty are discharged. Here, there was a good-faith (although mistaken) dispute as to whether the architect had produced defective plans. The "accord" was the client's offer to pay $7,500 for the settlement of the claim. "Satisfaction" of the accord occurred when the 364 architect accepted the payment. As a result, the original obligations under the contract were discharged, and the client is not liable for the remaining $2,500.

A seller entered into an agreement to sell a machine to a buyer for $5,000. At the time of the order, the buyer gave the seller a down payment of $1,000. The buyer then built a foundation for the machine at a cost of $250. The seller failed to deliver the machine. The buyer made reasonable efforts to find a similar machine and bought one for $5,500 that did not fit on the foundation. The buyer sued the seller for breach of contract. Which of these amounts claimed by the buyer, if any, could best be described as restitution? (A) The $250 cost of the foundation. (B) The $500 difference in price. (C) The $1,000 down payment. (D) None of the claimed amounts.

(C) is the best response, because restitution is designed to reverse unjust enrichment. A party's "restitution interest" is defined as his "interest in having restored to him any benefit that he has conferred on the other party." Rest. 2d of Contracts § 344(c). Of the three sums laid out by the buyer, the only one that took the form of a benefit conferred on the other party was the $1,000 down payment. Under UCC § 2-711 (1), if the seller fails to make delivery, the buyer is entitled to various items of damage "in addition to recovering so much of the price as has been paid!. " So in addition to such expectation-interest damages as the cost of cover, the buyer will be entitled to restitution of the down payment.

Collection of a debtor's $2,000 debt to a creditor was barred by the applicable statute of limitations. The debtor sold and delivered his car to a buyer under a written agreement, signed by the buyer, in which the buyer promised to pay the $2,000 purchase price to the creditor "in satisfaction of [the debtor's] debt to [the creditor!" Can the creditor recover the $2,000 from the buyer? (A) No, because payment of the $2,000 to the creditor would undermine the statutory public policy against enforcement of stale claims. (B) No, because the creditor's rights as an intended beneficiary are subject to any defenses available to the contracting parties between themselves. (C) Yes, because the buyer's promise to pay $2,000 to the creditor is enforceable by the creditor regardless of whether the debtor was legally obligated to pay the creditor anything (D) Yes, because the buver's promise to pay $2,000 to the creditor revived the uncollectible debt.

(C) is the best response, because the (7) the creditor is an intended beneficiary of the buyer's promise to pay $2,000 to the creditor; and (2) the buyer's promise would be interpreted as a promise to pay the specified amount (the $2,000) regardless of whether the debtor was legally obligated to pay that $2000 to the creditor. This is a third-party beneficiary problem: you're asked whether the beneficiary (the creditor) may recover against the promisor (the buyer) for the $2,000 that the promisor has promised the promise (the debtor) that the promisor will pay to the beneficiary (the creditor). The first question you have to answer is, was the beneficiary an "intended" beneficiary or an "incidental" beneficiary (to use the Second Restatement's phrasing)? You need to decide this because only an intended beneficiary - not an incidental beneficiary - may sue the promisor for enforcement of the promise. Let's look at how Rest. 2d, § 302 defines intended beneficiaries: § 302 Intended and Incidental Beneficiaries (1) Unless otherwise agreed between promisor and promise, a beneficiary of a promise is an intended beneficiary if recognition of a right to performance in the beneficiary is appropriate to effectuate the intention of the parties and either (a) the performance of the promise will satisfy an obligation of the promise to pay money to the beneficiary; or (b) the circumstances indicate that the promise intends to give the beneficiary the benefit of the promised performance (2) An incidental beneficiary is a beneficiary who is not an intended beneficiary.

A chef purchased the front portion of the land needed for a restaurant he desired to build and operate, but the back portion was the subject of a will dispute between a brother and his sister. The sister's attorney advised her that her claim was doubtful. The chef, knowing only that the unresolved dispute existed, agreed in a signed writing to pay the sister $6,000, payable $1,000 annually, in exchange for a quitclaim deed (a deed containing no warranties) from the sister, who promptly executed such a deed to the chef and received the chef's first annual payment. Shortly thereafter, the probate court handed down 320 a decision in the brother's favor, ruling that the sister had no interest in the land. This decision has become final. The chef subsequently defaulted when his second annual installment came due. In an action against the chef for breach of contract, the sister will probably

(C) is the best response, because the chef's promise of payment to the sister was supported by consideration. When the sister issued the quitclaim deed, she was effectively settling - by assigning to the chef - her claim against the property. So the question is whether one who promises to make payment in exchange for the surrender of an invalid claim has received consideration. (If the chef didn't receive consideration in return for his promise, that promise wouldn't be enforceable.) The answer is that the surrender of the claim that turns out to be invalid nonetheless constitutes consideration if either (1) the claim is in fact doubtful because of uncertainty as to the facts or the law; or (2) the surrendering party believes that the claim may be valid. Rest. 2d § 74(1). Here, (7) was the case (since the sister's attorney told her that her claim was doubtful, not that it was definitely invalid). Since the sister's surrender of the claim in a bargained-for exchange in return for the chef's promise of payment constituted consideration, the chef's promise is enforceable.

An employer offered to pay a terminated employee $50,000 to release all claims the employee might have against the employer. The employee orally accepted the offer. The employer then prepared an unsigned release agreement and sent it to the employee for him to sign. The employee carefully prepared, signed, and sent to the employer a substitute release agreement that was identical to the original except that it excluded from the release any age discrimination claims. The employer signed the substitute release without reading it. Shortly thereafter, the employee notified the employer that he intended to sue the emplover for age discrimination. Is the employer likely to prevail in an action seeking reformation of the release to conform to the parties' oral agreement? (A) No, because the employer acted unreasonably by failing to read the substitute release prior to signing it. (B) No, because the parol evidence rule will preclude evidence of the oral agreement. (C) Yes, because the employee's fraudulent behavior induced the employer's unilateral mistake. (D) Yes, because the parties were mutually mistaken regarding the contents of the signed release.

(C) is the best response, because the employee's failure to disclose the change he made to the release constituted misrepresentation, making reformation appropriate even though the employer did not read the release before signing it. For the employer to prevail, it will have to overcome three major obstacles. (1) the employer will have to show why the parol evidence rule does not exclude evidence of the oral agreement (which included a complete release of all claims); (2) even after the employer satisfies (1), it will have to show why it is entitled to reformation (which in this case will require it to show fraud I intentional misrepresentation) by the employee); and (3) the employer will have to show why its own failure to read the substitute release before signing it shouldn't deprive it of a reformation that it would otherwise be entitled to. But the employer will, in fact, be able to overcome all three obstacles, and will thus be entitled to reformation.

Pursuant to its current written contract with a boutique retailer of bath products, a soap maker was to supply 100 bars of soap each month. The contract called for payment upon delivery and specified that failure to paywould trigger an additional interest charge on the amount due. The retailer failed to timely pay for the first and second deliveries, and the soap maker imposed an additional interest charge of 10% of the amount due, which the retailer paid without objection. Under a previous contract between the retailer and the soap maker, which contained a similar clause, the soap maker had imposed an additional interest charge of 5% each of the three times that the retailer had failed to make a timely payment. However, in its dealings with other customers, the soap maker has imposed an interest charge of 6% of the amount due. There is no consistent practice among manufacturers of goods in general or soap in particular as to the rate of interest imposed on the unpaid amount due. This is true regardless of the type of retailer's boutique or otherwise. In a suit by the soap maker against the retailer for the unpaid amount due under the contract, what interest rate will the finder of fact likely determine formed part of the contract?

5%, the interest rate that reflects the most recent course of dealings between the soap maker and the retailer. 6%, the interest rate that reflects the course of dealings between the soap maker and its other customers. You Selected: 10%, the interest rate that reflects the course of performance under the current contract. A reasonable rate of interest, as there is no common trade practice upon which the court can rely.

A woman listed her house for sale with a real estate broker. The woman and the broker signed a listing agreement, under which the woman agreed that the broker was to be the woman's sole agent during the six-month period of the listing agreement. The listing agreement was not recorded. The list price for the house was $250,000. The broker produced a prospective buyer who offered to purchase the house for $249,000 and who was prequalified to purchase at that price. The woman rejected the offer, giving no reason for her rejection, and the listing agreement expired the next day. Three months later, the woman sold the house to a corporation for $249,000. The broker never had any contact with the corporation. Is the broker owed a commission?

A. No, because the listing agreement was not recorded. B. No, because the woman sold to a buyer found without the broker's help and after the expiration of the listing agreement. C. Yes, because the broker produced a ready, willing, and able buyer during the term of the listing agreement, and the buyer's offer matched the ultimate sale price. D. Yes, because the woman gave no reason for rejecting the prospective buyer's offer during the listing period. A real estate broker is someone typically employed by a seller to handle the sale of real property under a listing agreement. Most listing agreements provide for payment of commission if the broker procures a buyer who is ready, willing, and able to purchase the property on terms that are agreeable to the seller during the listing period. Therefore, a broker is generally not entitled to commission until a sales contract is executed OR, if the contract is subject to conditions, the conditions are met. The political-question doctrine bars actions that require courts to resolve issues concerning diplomatic recognition of a foreign government. However, courts have the power to determine interbranch disputes involving foreign affairs when they concern the validity of a federal statute.

An engineering firm submitted a bid to a municipality for the construction of a new wastewater treatment plant. The firm's bid included a subcontractor's bid to complete the electrical work on the plant for $100,000. The municipality awarded the construction contract to the firm. Later that day, before the firm told the subcontractor of the award, the subcontractor told the firm that it was withdrawing its bid because it had recently undertaken a new project that would absorb all its capacity for the next 18 months. The firm nevertheless accepted the subcontractor's bid and demanded that it perform the electrical work on the plant, but the subcontractor refused. The firm had to hire another subcontractor to perform the electrical work, at a cost of $115,000. The firm completed the construction of the plant at a profit. Which of the following statements correctly describes the firm's legal rights, if any, against the first subcontractor?

A. The firm is entitled to recover nominal damages only, because it completed the construction at a profit. B. The firm is entitled to recover reliance damages, because it detrimentally relied on the first subcontractor's bid when it submitted its own bid to the municipality. C. The firm is entitled to recover expectation damages, because the first subcontractor's bid was irrevocable for a reasonable time and the firm timely accepted it. D. The firm has no rights against the first subcontractor, because the first subcontractor was free to revoke its bid at any time before the firm accepted that bid. It true that an offeror generally has the power to revoke an offer up until the moment when the offeree accepts. But if the offeree detrimentally and reasonably relies on the offer's remaining at least temporarily open in a sub-contractor scenario like the one here, that reliance can make the subcontractor's offer temporarily irrevocable by transforming it into an option contract. That's what happened here: when the firm reasonably used the first subcontractor's bid as the basis for the firm's master bid, and then won the master contract, the firm was entitled to treat the sub's bid as temporarily irrevocable for long enough for the firm to accept that sub-bid.

A nature magazine advertised a photography contest in its January issue, offering "$1,000 to any subscriber who sends us a photograph of the rare Florida Grasshopper Sparrow that we use for the cover of our May issue. Only submissions meeting our technical specifications and received by April 1 will be considered." The only subscriber to respond to the advertised contest sent the magazine a photograph of the sparrow that met the magazine's technical specifications. The photograph arrived on March 15. However, due to an ecological disaster that occurred in early April, the magazine decided to use a different picture on the cover of its May issue. The magazine used the subscriber's picture on the cover of its June issue and has refused to pay $1,000 to the subscriber on the ground that it was not used on the May cover. Is the subscriber likely to prevail in a breach-of-contract action against the nature magazine?

Answers: No, because the subscriber's photograph was not used on the cover of the May issue. No, because the subscriber failed to adequately notify the magazine of his acceptance. You Selected: Yes, because all of the express conditions of the offer have been satisfied. Correct Answer: Yes, because the magazine prevented the publication of the photograph. When nonoccurrence of condition is excused Waiver Party waives condition by words or conduct Wrongful interference Party wrongfully prevents or interferes with condition's occurrence Estoppel Party indicates condition will not be enforced AND Other party reasonably & detrimentally relies on belief that condition has been waived Disproportionate forfeiture Party substantially performed & will be significantly harmed if condition is enforced

A homeowner entered into oral contracts with both a painter and a landscaper to perform services at his home. The landscaper was the first to begin the services, and shortly after he began to work, he realized that the projected cost of the project would increase dramatically. After the homeowner learned how high the cost of the landscaping services was going to be, he called the painter to tell her that he could not go through with their contract at that time. The painter stated that she had already purchased a standard set of paintbrushes to paint his home, as well as glass necessary to create a custom mosaic on a back corner of the house, according to the homeowner's specifications. She had also paid for a temporary city permit to park her utility van on the residential street where the homeowner lived. In a suit by the painter against the homeowner, which of the following is the painter LEAST likely to recover?

Answers: The contract price minus the market cost of performance. You Selected: The cost of the glass for the mosaic. Correct Answer: The cost of the paintbrushes. The cost of the parking permit. If such damages cannot be calculated with reasonable certainty, then the nonbreaching party may recover for any expenses incurred in reasonable reliancethat the contract would be performed—e.g., the cost of the glass for the mosaic and the parking permit (Choices B & D).* However, recovery of reliance damages may be reduced by the amount spent by the nonbreaching party on materials that could reasonably be repurposed for another job—e.g., the painter's paintbrushes. Therefore, the painter is least likely to recover that cost. *Note that a nonbreaching party cannot recover both expectation and reliance damages, so the painter can only recover the contract price or the price of the glass and the parking permit.

A homeowner entered into a written contract with a contractor to construct an elaborate tree house among the large trees located in the homeowner's backyard. After commencing construction of the tree house, the contractor discovered that one of the trees intended to be used as support for the tree house had a relatively common fungal infection in its core that would cause the strength of the tree's branches to falter if left untreated. Neither the homeowner nor the contractor had knowledge of the fungal infection when they entered into the contract, but the contractor knew that such infections were common in the area and did not request an inspection of the trees before entering the contract. The contractor also knew that treatment was available at a high cost, but that even after treatment, he would need to create additional heavy-load-bearing supports for the tree at a substantial cost. When the contractor informed the homeowner that he would not perform under the contract unless the homeowner provided at least 75% of the additional costs needed to make the structure safe, the homeowner refused to pay the additional amount. The homeowner then sued the contractor for breach of contract. What is the likely result?

Answers: The contractor wins, because his performance was discharged due to impracticability. The contractor wins, because neither party was aware of the fungal infection. Correct Answer: The homeowner wins, because the contractor assumed the risk of the fungal infection. You Selected: The homeowner wins, because the fungal infection did not render performance impossible.

Question 9588 A homeowner called a septic cleaning company and made arrangements for the company to remove the waste from the septic tank on the homeowner's property. After completing the job, the company mailed the homeowner a bill for $500, the fair market value of the services rendered by the company. The bill indicated that payment was due in 60 days. Upon receiving the bill, the homeowner called the company and informed it that, since he had lost his job due to an accident, he would not be paying the company's bill. The following day, the company filed suit for breach of contract. Ten days later, the homeowner moved to dismiss the suit. The court granted the motion, dismissing the suit without prejudice. Is the court's dismissal proper?

Answers: You Selected: No, because the parties' dealings created an implied-in-fact contract. No, because the homeowner has repudiated the contract. Yes, because the company failed to demand assurances. Correct Answer: Yes, because the company's complaint is premature. However, this doctrine does not apply when the date of performance has not passed and the nonrepudiating party has fully performed. Under those circumstances, the nonrepudiating party must wait until the repudiating party's performance is due before filing suit.

A produce wholesaler sent a written offer to a farmer to purchase all of the corn that the wholesaler required for his business from the farmer for a period of two years. Excited at the prospect of having a guaranteed sale for his corn, the farmer immediately communicated his acceptance to the wholesaler. The wholesaler and the farmer entered into a written contract reflecting the basic terms set forth in the wholesaler's offer. Six months after the contract was executed, the wholesaler determined that, while the farmer's corn was returning a profit, the farmer's corn was not selling as well as corn that the wholesaler could acquire from other sources. The wholesaler contacted the farmer and informed him that he no longer required any of the farmer's corn and would not be placing another order. The wholesaler immediately started buying his corn from another source. If the farmer sues the wholesaler for breach of contract, is he likely to prevail?

No, because the contract did not contain a specific quantity term. No, because the wholesaler no longer needed the farmer's goods. Yes, because the farmer relied on the wholesaler's promise. You Selected: Yes, because the wholesaler purchased corn from another source. Buyer's duty of good faith & fair dealing under requirements contract Buyer must purchase goods in question from seller only Requirements must approximate reasonably foreseeable figure Reasonable elasticity in requirements permitted (so long as any variations are in good faith) A requirements contract is an exclusive agreement between a buyer and a seller for the sale of as many goods as the buyer requires during a specified period. The buyer's purchase of the goods from another seller violates the implied duty of good faith and fair dealing and constitutes a breach of contract.

An heir hired an appraiser to appraise various items of personal property that she had inherited, including an original oil painting. The appraiser told the heir that he had no expertise in appraising art and recommended that she hire an art appraiser to value the painting. The heir, doubting that the painting was valuable, declined to follow the appraiser's advice and decided to sell the painting at a yard sale. She set the price at $100, assuming that this price reflected the painting's approximate value. A neighbor, who knew nothing about art, purchased the painting from the heir at the asking price. The neighbor and the heir later discovered that the painting was worth over $900,000. Would the heir be likely to prevail in an action to rescind the contract?

No, because the heir bore the risk of any mistake (7 as to the true value of the painting. No, because the neighbor possessed no specialized knowledge of the true value of the painting. Yes, because enforcement of the contract would be unconscionable, given its disadvantageous impact on the heir. Yes, because the parties were mutually mistaken as ( to the true value of the painting at the time of the sale. Mutual mistake occurs when both parties enter into a contract based on the same mistake. In such cases, the contract may be voidable by the adversely affected party if: • the mistake relates to a basic assumption of the contract • the mistake materially affects the agreed-upon exchange of performances (ie, the adversely affected party cannot fairly be required to carry it out) and • the adversely affected party did not assume (ie, bear) the risk of the mistake. One way that a party assumes the risk of a mistake is by proceeding with conscious ignorance (ie, awareness of his/her limited knowledge) of the facts related to the mistake.

A library contacted a local artist expressing an interest in purchasing a particular one of the artist's sculptures for display at the library. The library's agent and the artist executed a written contract that was signed by both parties and provided that the library would purchase the sculpture for $1,000 due upon delivery of the sculpture to the library. Just before they signed the contract, the agent told the artist, "Plan on delivering the sculpture in 10 days, but please remember that the library's obligation to purchase the sculpture will be conditioned on the approval of the chairperson of the Artistic Patronage Council, as it will be providing the library with the funds for this sale." The chairperson of the Artistic Patronage Council orally approved the sale the next day. However, 10 days after the contract was executed, the artist decided that he did not want to sell the sculpture. If the library sues the artist for breach of contract, is the library likely to prevail?

No, because the library's agent made an illusory promise. No, because there was no mutuality of remedy when the contract was executed. You Selected: Yes, because the agreement was supported by good consideration even though it was conditioned on an uncertain event. Yes, because the artist waived any lack of consideration by signing the contract. To be enforceable, a contract must generally be supported by valuable consideration—i.e., a bargained-for change in the legal position between the parties. Performance under a contract may be conditioned upon a condition precedent (which delays performance) or a condition subsequent (which excuses performance).

On October 1, a retail sporting equipment store telephoned a shoe manufacturer and offered to buy a minimum of 50 and a maximum of 100 pairs of running shoes at $40 a pair to be delivered in 60 days. The manufacturer orally accepted the offer and then immediately faxed a signed letter to the store. The letter contained the following language: "This letter confirms our agreement by telephone on October 1 to sell you 50 pairs of running shoes for 60-day delivery." Sixty days later, the manufacturer delivered 100 pairs of conforming running shoes to the store. However, the store rejected all 100 pairs of shoes because it had found the same shoes from another seller for $30 a pair. Due to lack of demand, the manufacturer cannot resell the 100 shoes without suffering an economic loss. Can the manufacturer enforce a contract against the store?

No, because the manufacturer did not state the agreed-upon price term in the faxed letter. No, because the store did not sign the faxed letter. You Selected: Yes, for the sale of 50 pairs of shoes, because the manufacturer's faxed letter stated that quantity term. Yes, for the sale of 100 pairs of shoes, because the store wrongfully rejected that quantity of properly delivered and conforming shoes. Here, the store offered to purchase 50 to 100 pairs of shoes from the manufacturer. The manufacturer's acceptance contained a new term for the sale of 50 pairs of shoes. Since both parties are merchants, the acceptance was effective despite the new term and that term became part of the contract since none of the bulleted exceptions apply. *In contrast, a revised term is one that conflicts with or contradicts a term in the offer.

A restaurant owner sent a signed order form to a produce supplier that read: "Please ship us 100 pounds of potatoes at your current price." The supplier received the order form onNovember 9. Later the same day, the supplier mailed the owner a letter with the proper address and postage. The letter stated that the order had been accepted at the supplier's current price for potatoes. On November 10, before receiving the supplier's reply, the owner telephoned the supplier to inform the supplier that the owner had found a closer supplier and was canceling the order. The supplier asked the owner to reconsider, but the owner refused. On November 11, the owner received the supplier's letter. Relying on the owner's telephone call, the supplier never shipped any potatoes to the restaurant. As of November 12, is there an enforceable contract between the supplier and the owner?

No, because the owner effectively revoked its offer before it was properly accepted. No, because the owner's offer could be accepted only by shipment of the goods, and the supplier never shipped the potatoes. Yes, because the owner's offer was irrevocable for a reasonable time. You Selected: Yes, because the supplier effectively accepted the owner's offer before the owner revoked it. An acceptance that is mailed within the allotted response time is effective when sent rather than upon receipt, unless the offer provides otherwise. The mailing must be properly addressed and include correct postage. Here, those requirements are met. Offers revoked by the offeror are effective upon receipt. Therefore, the supplier effectively accepted the offer before receiving the owner's revocation of the offer, and a contract was formed upon acceptance

On November 1, the owner of a yacht posted a flyer at a local coffee shop reading, "Yacht for Sale: Make me an offer!" The flyer also included the owner's phone number. A buyer called the owner on November 3 to ask how much the owner wanted for the yacht. The owner said, "Well, I'd hate to part with it for less than $55,000, but if you can pay me $50,000 by November 20, I'd sell it to you. I'll hold onto the yacht for you until then." Elated, the buyer took steps to obtain a loan by November 20. On November 15, a second buyer called the owner and offered to buy the yacht for $60,000. The owner immediately accepted, and the second buyer picked up the yacht the next day. On November 20, having obtained a loan, the first buyer visited the owner with a check for $50,000. The first buyer then learned the owner had already sold the yacht. Can the first buyer bring a successful suit against the owner for breach of contract?

No, because the owner's statement to the first buyer was only an invitation to deal. No, because the second buyer offered more money for the yacht than the first buyer agreed to pay. Yes, because the owner promised to keep the offer open for a specific period of time. You Selected: Yes, because the owner's offer to the first buyer was still outstanding on November 20. An offer can be accepted at any time before the offer is revoked. The offeror can revoke the offer by manifesting an intent not to enter into the proposed contract. This can occur in two ways: expressly - when the offeror communicates the revocation directly to the offeree constructively - when the offeree acquires reliable information that the offeror has taken definite action inconsistent with the offer Here, the owner never revoked his offer.

A law student attended law school on a full scholarship. At the end of the law student's second year, she lost her scholarship. In order to fund her third year, she borrowed $50,000 from her rich uncle. They executed a written agreement stating that the law student would repay the loan two years from May 15, the date of her law school graduation. On May 15, two years after her graduation, the law student did not pay her uncle back because she had been unable to find a job as a lawyer. Instead, she was working as a server at a coffee shop. The uncle took no legal action. Four years later, the law student was still unable to pay the uncle back, but she did write him a signed letter stating that "I know I still owe you $50,000. I will repay you $50,000 if I get a law firm job." The statute of limitations for suits to collect debts in the jurisdiction is three years. Is the law student's promise contained in the letter to repay the loan enforceable?

No, because the promise to repay is not supported by consideration. No, because the statute of limitations for collecting debts in the jurisdiction is three years. You Selected: Yes, because the promise was made after the statute of limitations had run. Yes, because the uncle's foregoing of legal action constitutes consideration.

A maker of perfume contacted a manufacturer about supplying 1,000 readily available glass bottles for retail sales of perfume. The manufacturer offered to supply the bottles and to ship them within one week. The perfumer responded, "Ship them as soon as possible." The manufacturer shipped 1,000 bottles to the perfumer five days later. The perfumer accepted the bottles and filled them with perfume. Without waiting for the manufacturer's invoice, the perfumer sent a payment to the manufacturer based on a price of $2.50 per bottle. Prior to receiving this payment, the manufacturer sent the perfumer an invoice that reflected a charge of $3.50 per bottle. When the perfumer refused to pay $3.50 per bottle, the manufacturer returned the payment to the perfumer and initiated an action for the price. The court determined that a reasonable price for the bottles at the time of delivery was $3.25 per bottle. What amount should the court award the manufacturer per bottle?

Nothing, because no contract was formed in the absence of a price term. $2.50, because the perfumer, as offeror, was master of the offer. You Selected: $3.25, because this was a reasonable price for the bottles at the time of delivery. $3.50, because the manufacturer, as offeror, was master of the offer. if a contract omits a price term—or if the parties agree to set the price in the future but fail to do so—then the UCC supplies a reasonable price at the time of delivery.

A construction company contracted with a manufacturer to purchase 100 identical prefabricated windows to use while constructing houses in a gated community. The windows were to be delivered in shipments of 25 windows each on April 1, May 15, July 1, and August 15. The written contract, signed by both parties, was silent as to when payment for each shipment would be due. The manufacturer made the first two shipments in conformity with the contract requirements, and the construction company paid one-fourth of the full contract price upon each delivery. However, on June 1, the manufacturer demanded that the construction company pay the entire remainder of the contract price before the manufacturer made any further shipments. Which of the following statements is true?

The construction company has no duty under the contract to make any payments until the final delivery is made. You Selected: The construction company must pay the manufacturer one-fourth of the contract price upon delivery of each conforming shipment of windows. The construction company's failure to pay the requested sum will amount to a repudiation of the contract. The manufacturer waived his right to demand immediate payment of the full contract price when he accepted the first payment of one-fourth of the contract price on April 1. Special rules apply to installment contracts for the sale of goods (e.g., windows), which are governed by the UCC. Under the UCC, an installment contract is defined as a contract in which the goods are to be delivered in multiple shipments, and each shipment is to be separately accepted by the buyer. Payment by the buyer is due upon each delivery unless the price cannot be apportioned.

A licensing agreement provided that a manufacturer could use an inventor's patent in manufacturing its products for 10 years. Immediately thereafter, the inventor assigned his rights to receive payments pursuant to the licensing agreement to a corporation. The inventor did not receive compensation for this assignment. The inventor, upon his death five years later, devised his stock in the corporation to his daughter and all of his remaining property to his son. To whom should the manufacturer make its payments under the licensing agreement?

The corporation. The inventor's daughter. You Selected: The inventor's son. No one, because the manufacturer's obligation to make payments under the licensing agreement terminated upon the death of the inventor. An assignment is the transfer of contractual rights to a third party. If an assignment is not supported by consideration, then it is a gratuitous assignment and is generally revocable (exceptions listed in the table above). A revocable assignment is automatically revoked upon the death, incapacity, or bankruptcy of the assignor. the inventor assigned his right to receive payments under a licensing agreement to the corporation without receiving compensation or other consideration. Therefore, the assignment was automatically revoked upon the inventor's death, and the right to receive payment returned to his estate

A jeweler and a goldsmith signed a written agreement that provided as follows: "For $3,000, the goldsmith shall sell to the jeweler a size six gold ring setting that the jeweler shall select from only the goldsmith's white gold ring designs." The agreement did not address any other specific terms with regard to the business arrangement between the jeweler and the goldsmith. When the jeweler arrived to select a ring, he refused to select one of the goldsmith's white gold ring designs. The jeweler claimed that the goldsmith, immediately prior to the execution of the written agreement, had orally agreed to broaden the jeweler's choices to also include rose gold ring designs. The jeweler also claimed that the goldsmith had, at the same time, orally agreed to include a set of earring settings, valued at $1,000, as an incentive for the jeweler's continued business. The goldsmith refused to sell to the jeweler any of his rose gold ring designs or include the earring settings. If the jeweler sues the goldsmith for damages, how should the court handle the evidence of the alleged oral agreements?

The court should admit the evidence as to both the promise to include the earring settings and the option to choose a rose gold ring design. Correct Answer: The court should admit the evidence as to the promise to include the earring settings but not the option to choose a rose gold ring design. The court should admit the evidence as to the option to choose a rose gold ring design but not the promise to include the earring settings. You Selected: The court should exclude the evidence as to both the option to choose a rose gold ring design and the promise to include the earring settings. Educational Objective:The UCC presumes that a written contract is partially integrated. As a result, evidence that supplements the written contract is admissible—but evidence that contradicts the writing is inadmissible—under the parol evidence rule.

A farmer and an innkeeper have dealt with each other on a weekly basis for the last five years. Every Sunday, the innkeeper sends a request for his produce requirements for the following week, and the farmer delivers the requested produce to the inn the following Saturday evening. A few days before delivery, the innkeeper signs an invoice indicating the agreed-upon price for that week's delivery and setting the deadline for timely payment as six weeks after delivery. Each invoice signed by the innkeeper in the last five years has noted that the farmer does not offer a discount for predelivery payments, but the invoices are silent regarding discounts for cash payments. The custom of the industry in the jurisdiction is to allow a 5% discountfrom the invoice price for any payments made in cash and an additional 5% discount for early payments made before delivery. However, in all of their prior transactions, the innkeeper paid in cash and always deducted 5% from each invoice price. The innkeeper has never paid before delivery, and the farmer has never objected to the 5% discount. The present contract called for a single delivery of produce for $1,000, and the same delivery procedures and invoice were used in the present contract as had been used previously. The innkeeper paid the farmer $900 in cash the day before the delivery, taking a 10% discount. However, the day after delivery, the innkeeper received the following letter from the farmer: "Your payment of $900 is insufficient under our contract. Payment in full in accordance with your signed invoice is due within six weeks of delivery. Recall that no discounts are permitted under our contract." Which of the following statements concerning these facts is most accurate?

The custom of the trade controls, and the innkeeper does not owe the farmer any additional payment. The express contract terms control, and the innkeeper owes the farmer $100. You Selected: The parties' course of dealing controls, and the innkeeper owes the farmer $50. The parties' course of performance controls, and the innkeeper owes the farmer $50. Under the UCC parol evidence rule, express contract terms can be supplemented or explained with evidence of course of performance, course of dealing, or trade usage (in that order of priority). However, this evidence cannot be used to contradict the express terms of the contract. here is no course of performance here because this single-delivery contract does not involve repeated occasions for performance (Choice D). But there is a course of dealing and trade usage, both of which provide a 5% discount for cash payments. Since course of dealing has priority over trade usage, course of dealing controls (Choice A). Therefore, the innkeeper was entitled to only a 5% discount for the cash payment and owes the farmer $50.

A dairy ordered a tank trailer from a commercial retailer. The contract did not identify a specific tank trailer, but did specify various features that the tank trailer should have. The contract called for the retailer to deliver the tank trailer to the dairy. Subsequently, the retailer acquired a tank trailer from a manufacturer that it believed met the requirements of the contract. Several days later, the retailer, in accordance with the contract, had the dairy's emblem painted on each side of the tank trailer, and then delivered it to the dairy. The tank trailer did not, in fact, conform to the contract. At which time did the dairy acquire an insurable interest in the tank trailer?

When the contract was entered into. When the retailer acquired the tank trailer from the manufacturer. Correct Answer: When the retailer had the dairy's emblem painted on the tank trailer. You Selected: When the retailer delivered the tank trailer. A buyer acquires an insurable interest in goods upon the identification of the goods. Where the contract is for future goods (i.e., goods that are not both existing and identified), the buyer does not acquire an insurable interest until the seller designates goods as those to which the contract refers, unless the parties have explicitly agreed otherwise. Since the dairy did not select a tank trailer from the retailer's inventory or otherwise select a specific trailer, the contract was for the sale of future goods. By painting the dairy's emblem on the tank trailer, the seller effectively designated that tank trailer as the dairy's. Consequently, at that time, the dairy acquired an insurable interest in it.

At the beginning of the month, an aunt called her niece who lived in a distant city. During the conversation, the aunt promised to give a family heirloom worth $50,000 to her niece if the niece came to the aunt's home to retrieve it. The niece promised to come. The following day the niece bought an airline ticket to fly to the city where her aunt lived at the end of the month. The day before the niece was to make the trip, her aunt died. Under the terms of the aunt's will, the heirloom was left to someone else. Can the niece acquire the heirloom by enforcing her aunt's promise against the aunt's estate?

Yes, under the doctrine of promissory estoppel. Yes, because there was an exchange of promises. No, because the aunt's promise was oral. No, because the aunt promised to make a gift. A promise to make a gift is generally unenforceable because it does not involve the bargained-for exchange that is required to constitute valuable consideration. Under the doctrine of promissory estoppel, a party that reasonably and detrimentally relies on another party's promise may recover the costs of relying on that promise—but not the value of the promise itself. Therefore, the niece would be able to recover the cost of the airplane ticket she bought in reliance on the aunt's promise to give her the heirloom—but not the heirloom itself.

On January 8, a liquor company sent a signed, written offer to a retailer containing the following: "We will sell you our last 500 cases of our caffeine-infused vodka for $100 per case." Upon receiving the liquor company's offer, the retailer decided to take a couple of days to contemplate the offer. On January 15, after not hearing from the retailer, the liquor company decided it was no longer willing to sell the vodka to the retailer, but it did not contact the retailer. On January 17, the retailer decided that it was not interested in the vodka and placed a letter rejecting the offer in the mail. However, after hearing about the popularity of the vodka from a number of patrons that same day, the retailer immediately mailed a signed acceptance to the liquor company. The liquor company received the retailer's acceptance on January 20. Later that same day but unbeknownst to either party, the state enacted a law prohibiting the sale or distribution of the caffeine-infused vodka due to a number of serious health risks connected to the product. Due to an error at the post office, the retailer's initial rejection was not received until January 21. The retailer and the liquor company both learned of the state law on January 22. Which of the following is an accurate statement regarding the relationship between the retailer and the liquor company as of January 23?

You Selected: A contract was formed on January 20, but the duty to perform under the contract is discharged due to illegality. An enforceable contract was formed on January 20. No contract was formed because the liquor company's offer was revoked on January 15. No contract was formed because the retailer's rejection was effective on January 17.

A chemistry professor offered to sell her colleague an autographed first edition of a novel for $1,000. The professor provided her colleague with a signed written statement specifying the terms of the offer and stating that the offer would remain open for one week. Two days later, the colleague learned that the professor had sold the book to someone else in their department. The next day, the colleague showed up at the professor's office with $1,000, asking to purchase the book. The professor apologized, saying that the book had already been sold. Is the colleague likely to succeed in an action for breach of contract?

You Selected: No, because an option contract is not valid unless the offeror is a merchant. Correct Answer: No, because the colleague learned that the book had been sold before accepting the offer. Yes, because the professor did not revoke the offer prior to the colleague's acceptance. Yes, because the offer was contained in a signed writing and thus could not be revoked.

A caterer contracted with a local farmer for the delivery of three dozen fresh local eggs. The contract provided that because the caterer planned to use the eggshells to serve one of her signature dessert recipes, the eggs needed to be a uniform color. The farmer delivered the caterer 20 white eggs and 16 speckled eggs. The caterer immediately emailed the farmer and informed him that she was rejecting the eggs because she could not use the inconsistent shells to serve her desserts. The caterer also told the farmer that she did not have the ability to refrigerate the eggs or the space to store them for long and that she would wait for his instructions. The caterer stored the eggs on her countertop for a week and had not heard from the farmer. Concerned that the unrefrigerated eggs would soon spoil, the caterer promptly returned the eggs to the farmer. Due to the perishable nature of the eggs, the farmer had to resell the eggs at half the normal price. If the farmer brings a breach-of-contract claim against the caterer to recover the full contract price of the eggs, will he succeed?

You Selected: No, because the caterer behaved appropriately after rightfully rejecting the eggs. No, because the caterer had no obligations regarding the nonconforming eggs. Yes, because the caterer had a duty to retain the eggs until the farmer retrieved them. Yes, because the caterer was required to sell the eggs on the farmer's behalf. After rejection, the buyer has an obligation to take reasonable care of any goods in its possession until the seller has had a reasonable amount of time to retrieve them. When, as here, the seller does not retrieve the goods or provide further instructions, the buyer may generally choose to store, reship, or sell the goods on the seller's behalf. However, the buyer is required to sell the goods on the seller's account if: The buyer is a merchant—i.e., one who regularly deals in goods of the kind involved or who, by occupation, holds him/herself out as having knowledge or skills unique to the goods involved* the goods involved are perishable or threaten to speedily decline in value and the seller has no local agent to whom the goods can be returned.

The owner of a ferry boat operated the boat only during daylight hours during the summer months of June, July, and August. On March 1, the owner entered into a written agreement with a man to serve as the captain of the boat for the upcoming season. On May 1, the owner contracted with a woman to serve as the captain of the boat. On May 30, the man was diagnosed with an illness, and the treatment for this illness prevented him from being employed until the following year. On May 31, the owner learned of the man's illness and told the man not to worry about their contract as he had found someone else to serve as captain of the boat. The woman served as captain of the boat for the summer months of June, July, and August that year. On September 1, the man sued the owner for damages based on a breach of their contract. Can the man recover damages based on breach of contract?

You Selected: No, because the man was unable to serve as the captain of the boat during the summer months. No, because the owner informed the man about the owner's contract with the woman prior to June 1. Yes, because the owner did not inform the man of the owner's contract with the woman until after the owner learned of the man's illness. Yes, because the owner's contract with the woman constituted an anticipatory breach of the owner's contract with the man. A nonrepudiating party who materially breaches the contract cannot recover damages for the other party's anticipatory breach because the material breach discharges the other party's duty to perform.

A men's apparel wholesaler was trying to expand its business, so it reached out to an online company that sold men's ties. The wholesaler mailed a letter to the online company offering to sell it 1,000 silk ties at a wholesale price of $15 per tie. The signed letter, dated July 1, assured the online company that the option to purchase would stay open but did not specifically state how long the option would remain open. A year later, the online company sent a letter to the wholesaler accepting its offer to sell the company 1,000 silk ties at a wholesale price of $15 per tie. The wholesaler and online company had no prior dealings, and offers of this kind in the industry generally do not remain open for a year with no further contact between the parties. If no other correspondence or action was taken by either party between the wholesaler's offer and the online company's purported acceptance, is there an enforceable contract between the parties?

You Selected: No, because the online company did not accept the offer within a reasonable period of time. No, because the online company did not give consideration to keep the offer open. Yes, because the time period of irrevocability was not stated. Yes, because the wholesaler's offer had no termination date.

The owner of a rare eighteenth-century chest offered to sell it to a connoisseur of antiques for $75,000. The connoisseur countered that she would buy the chest for $50,000. The owner rejected this price. The owner and the connoisseur then executed a written agreement for the sale of the chest at a price to be determined only by a particular antiques dealer whose expertise in valuing this rare item they both trusted. Two weeks later, the agreed-upon antiques dealer examined the chest. He told the owner and the connoisseur that he had to do further research on the chest but that he would let them know his decision in several days. Unfortunately, the dealer died before doing so. A reasonable price for the chest can be established by the court. Is there likely an enforceable contract?

You Selected: No, because the owner and the connoisseur did not intend to be bound unless the dealer set the price of the chest. No, because the price of the chest was not determined at the time the agreement was executed. Yes, because a reasonable price for the chest can be established by the court. Yes, because the owner and the connoisseur executed a written agreement for the sale of the chest. Under the UCC, a contract for the sale of goods is formed if both parties intend to contract and there is a reasonably certain basis for giving a remedy in the event of a breach. Intent to contract is judged by outward, objective manifestations of intent, as interpreted by a reasonable person. So when an agreement reflects an intent to be bound only if the price is subsequently set, no contract is formeduntil the price is set.


Ensembles d'études connexes

Chapter 1: The basics of Nutrition

View Set

PHYSICAL SCIENCE - Chapter 3 Quiz

View Set

Chapter 16: Auditing Operations and Completing the Audit

View Set

Trail Guide Spine and Thorax review questions

View Set

Lección 14 ¿Cómo pagas? ; Tu empresa

View Set

1. Introduction to C++ (Great Courses)

View Set